120
LECTURE 1-5 ANALYTICAL REASONING Questions 1- 3 Three men (Tom, Peter and Jack) and three women (Eliza, Anne and Karen) are spending a few months at a hillside. They are to stay in a row of nine cottages, each one living in his or her own cottage. There are no others staying in the same row of houses. 1. Anne, Tom and Jack do not want to stay in any cottage, which is at the end of the row. 2. Eliza and Anne are unwilling to stay besides any occupied cottage.. 3. Karen is next to Peter and Jack. 4. Between Anne and Jack's cottage there is just one vacant house. 5. None of the girls occupy adjacent cottages. 6. The house occupied by Tom is next to an end cottage. 1. Which of the above statements can be said to have been derived from two other statements ? A. Statement 1 B. Statement 2 C. Statement 3 D. Statement 5 E. Statement 6 Ans : D 2. How many of them occupy cottages next to a vacant cottage ? A. 2 B. 3 C. 4 D. 5 E. 6 1

msd1

Embed Size (px)

Citation preview

Page 1: msd1

LECTURE 1-5

ANALYTICAL REASONING

Questions 1- 3

Three men (Tom, Peter and Jack) and three women (Eliza, Anne and Karen) are spending a few months at a hillside. They are to stay in a row of nine cottages, each one living in his or her own cottage. There are no others staying in the same row of houses.

1. Anne, Tom and Jack do not want to stay in any cottage, which is at the end of the row.

2. Eliza and Anne are unwilling to stay besides any occupied cottage.. 3. Karen is next to Peter and Jack. 4. Between Anne and Jack's cottage there is just one vacant house. 5. None of the girls occupy adjacent cottages. 6. The house occupied by Tom is next to an end cottage.

1. Which of the above statements can be said to have been derived from two other statements ?

A. Statement 1 B. Statement 2 C. Statement 3 D. Statement 5 E. Statement 6

Ans : D

2. How many of them occupy cottages next to a vacant cottage ? A. 2 B. 3 C. 4 D. 5 E. 6

Ans : C

3. Which among these statement(s) are true ?

I. Anne is between Eliza and Jack. II. At the most four persons can have occupied cottages on either side

of them. . III. Tom stays besides Peter.

A. I only

1

Page 2: msd1

B. II only C. I and III only D. II and III only E. I, II and III

Ans : C

Questions 4 - 7

An employee has been assigned the task of allotting offices to six of the staff members. The offices are numbered 1 - 6. The offices are arranged in a row and they are separated from each other by six foot high dividers. Hence voices, sounds and cigarette smoke flow easily from one office to another.

Miss Robert's needs to use the telephone quite often throughout the day. Mr. Mike and Mr. Brown need adjacent offices as they need to consult each other often while working. Miss. Hardy, is a senior employee and has to be allotted the office number 5, having the biggest window. .

Mr. Donald requires silence in the offices next to his. Mr. Tim, Mr. Mike and Mr. Donald are all smokers. Miss Hardy finds tobacco smoke allergic and consecutively the offices next to hers to be occupied by non-smokers.

Unless specifically stated all the employees maintain an atmosphere of silence during office hours.

4. The ideal candidate to occupy the office furthest from Mr. Brown would be A. Miss Hardy

B. Mr. Mike

C. Mr. Tim

D. Mr. Donald

E. Mr. Robert

Ans : D

5. The three employees who are smokers should be seated in the offices. A. 1, 2 and 4 B. 2, 3 and 6 C. 1, 2 and 3 D. 1, 2 and 3 E. 1, 2 and 6

2

Page 3: msd1

Ans : D

6. The ideal office for Mr. Mike would be. A. 2 B. 6 C. 1 D. 3 E. 4

Ans : D

7. In the event of what occurrence, within a period of one month since the assignment of the offices, would a request for a change in office be put forth by one or more employees ?

A. Mr. Donald quitting smoking. B. The installation of a noisy teletype machine by Miss Hardy in her

office. C. Mr. Robert's needing silence in the office (s) next to her own. . D. Mr. Brown suffering from laryngitis. E. Mr. Tim taking over the duties formerly taken care of by Miss.

Robert. .

Ans : E

In an experiment conducted at a laboratory, 160 white mice were injected with Serum D. 160 other white mice were injected with a harmless sugar solution .In two weeks time 39% of the white mice, who were injected with Serum D contracted the highly contagious and often fatal disease, jungle fever. Hence, it can be concluded that jungle fever is caused by some elements similar to the elements in Serum D.

8. The above discussion would be weakened most severely in case it is shown that

A. People contracting jungle fever are usually the victims of the bite of the South American Lesser Hooded Viper.

B. One among the 160 white mice had already contracted jungle fever prior to the laboratory experiment.

C. The natural habitats of white mice does not contain any of the elements found in Serum D.

D. The scientists administered the injections being ignorant of the contents of the solutions used.

E. The 160 white mice used in the laboratory experiment were kept isolated from each other.

Ans : B

3

Page 4: msd1

9. The above argument would be highly empowered in case it were shown that:

A. Some of the elements in Serum D are extracted from the root of a certain poisonous jungle wildflower.

B. Within a period of two weeks about 40% of the white mice, who were injected with a harmless sugar solution also contracted jungle fever.

C. Almost all the white mice died within a period of two days after the first symptoms appeared.

D. Normally the rate of jungle fever among white mice is less than 0.01%.

E. Invariably the blood of the victims of jungle fever victims contains a high level of a certain toxic substance also found in serum D.

Ans : E

10.Distribution of leaflets and delivering speeches on government property should be outlawed. Radicals and fanatics have no right to use public property when peddling their unsavory views.

The argument above is based on the postulate

A. The general public has a special concern in the free exchange of different political views.

B. Radicals and fanatics prefer the use of public property while propagating their viewpoint.

C. Every person who hands out leaflets and delivers speeches is a radical or fanatic.

D. Legal constraints which are applicable to one group need not be equally applicable to all.

E. Any political activity, which hinders the proper functioning of the government should not be protected by the law.

Ans : C

Questions 11 - 12

Successfully adjusting to one's environment leads to happiness. War at a universal level war destroys the weaker people, who are the most unable to adjust to their environment. Thus, war at the universal level puts weaklings out of their misery and allows more space for their predators to enjoy life in a better manner. As those actions have to be performed, which maximize the level of happiness of the greatest number, war at a universal level should take place.

11.What response would the author of the above discussion come up with, in the case of the objection that the weaklings far exceed strong people?

4

Page 5: msd1

I. He would respond with the statement that the person making the objection is a weakling.

II. He would respond by saying that weaklings will be miserable no matter what happens.

III. He would respond with the statement that the strong would be frustrated if the weaklings are destroyed.

A. I only B. II only C. III only D. I and II only E. II and III only

Ans : E

12.The author's discussion would be greatly if he agreed to which of the following?

I. Technology could change the environment. II. War at the universal level would be an integral part of the

environment. III. It is possible for the strong to survive without suppressing the weak.

A. I only B. II only C. III only D. I and III only E. I, II and III only

Ans : A

Questions 13 - 15

Come back with us to the real America leaving behind the turmoil of civilization. The real America is still inhabited by the eagle, the buffalo, the mountain lion and elk; it is still spacious, sprawling and majestic. Experience the freedom and serenity still to be found in

13.Choose the best option to complete the above statement: A. the natural beauty of our land B. the fascinating urban centers C. the wild terrain of Africa D. one's own subconscious E. the great sprawling cities of the Southwest

Ans : A

5

Page 6: msd1

14.The above paragraph is most likely to appear in which of the following? A. A Hunter's Guide to The United States B. Exploring the Great Outdoors C. The Quiet Beauty of Alaska D. How the Eagle Became Extinct E. Returning to America

Ans : D

15.When I am elected, I will work towards effecting those changes for which I have been fighting all these years. We will work together to do away with the bureaucratic bogs which have existed ever since my opponent took office. Everyone of you knows what I stand for; I invite my opponent to ...

For completion of the above statement choose the best option:

A. hand in his resignation graciously B. make his stance clear C. stop lying to the public D. get our city more federal aid E. extend his support to me

Ans : B

16.We can never make our beliefs regarding the world certain. Even scientific theory of a most rigorous and well-confirmed nature is likely to change over a decade or even tomorrow. If we refuse to even try to understand, then it is like resigning from the human race. Undoubtedly life of an unexamined kind is worth living in other respects--as it is no mean thing to be a vegetable or an animal. It is also true that a man wishes to see this speculative domain beyond his next dinner.

From the above passage it is clear that the author believes that

A. men would not do well to speculate B. progress in the scientific field is impossible C. one should live life with the dictum 'what will be will be' D. men should ignore their animal needs E. men are different from animals as far as their reasoning abilities are

concerned.

Ans : E

Questions 17-21

6

Page 7: msd1

Two or more essences out of a stock of five essences-- L, M, N, O, and P are used in making all perfumes by a manufacturer. He has learned that for a blend of essences to be agreeable it should comply with all the rules listed below.

A perfume containing L, should also contain the essence N, and the quantity of N should be twice as that of L. A perfume containing M, must also have O as one of its components and they should be in equal proportion. A single perfume should never contain N as well as O. O and P should not be used together. A perfume containing the essence P should contain P in such a proportion that the total amount of P present should be greater than the total amount of the other essence or essences used.

17.Among the following which is an agreeable formula for a perfume? A. One part L, one part P B. Two parts M, two parts L C. Three parts N, three parts L D. Four parts O, four parts M E. Five parts P, five parts M

Ans : D

18.Adding more amount of essence N will make which of the following perfumes agreeable?

A. One part L, one part N, five parts P B. Two parts M, two parts N, two parts P C. One part M, one part N, one part P D. Two parts M, one part N, four parts P E. Two parts N, one part O, three parts P

Ans : A

19.Among the following, the addition of which combination would make an unagreeable perfume containing two parts N and one part P agreeable?

(A) One part L (B) One part M (C) Two parts N (D) One part O (E) Two parts P

Ans : E

20.Among the following which combination cannot be used together in an agreeable perfume containing two or more essences?

A. L and M B. L and N C. L and P

7

Page 8: msd1

D. M and O E. P and N

Ans : A

21.Among the below mentioned formulas, which can be made agreeable by the eliminating some or all of one essence ?

A. One part L, one part M, one part N, four parts P B. One part L, two parts N, one part O, four parts P C. One part L, one part M, one part O, one part P D. Two parts L, two parts N, one part O, two parts P E. Two parts M, one part N, two parts O, three parts P

Ans : B

Questions 13 - 15

Come back with us to the real America leaving behind the turmoil of civilization. The real America is still inhabited by the eagle, the buffalo, the mountain lion and elk; it is still spacious, sprawling and majestic. Experience the freedom and serenity still to be found in

13.Choose the best option to complete the above statement: A. the natural beauty of our land B. the fascinating urban centers C. the wild terrain of Africa D. one's own subconscious E. the great sprawling cities of the Southwest

Ans : A

14.The above paragraph is most likely to appear in which of the following? A. A Hunter's Guide to The United States B. Exploring the Great Outdoors C. The Quiet Beauty of Alaska D. How the Eagle Became Extinct E. Returning to America

Ans : D

15.When I am elected, I will work towards effecting those changes for which I have been fighting all these years. We will work together to do away with the bureaucratic bogs which have existed ever since my opponent took office. Everyone of you knows what I stand for; I invite my opponent to ...

For completion of the above statement choose the best option:

8

Page 9: msd1

A. hand in his resignation graciously B. make his stance clear C. stop lying to the public D. get our city more federal aid E. extend his support to me

Ans : B

16.We can never make our beliefs regarding the world certain. Even scientific theory of a most rigorous and well-confirmed nature is likely to change over a decade or even tomorrow. If we refuse to even try to understand, then it is like resigning from the human race. Undoubtedly life of an unexamined kind is worth living in other respects--as it is no mean thing to be a vegetable or an animal. It is also true that a man wishes to see this speculative domain beyond his next dinner.

From the above passage it is clear that the author believes that

A. men would not do well to speculate B. progress in the scientific field is impossible C. one should live life with the dictum 'what will be will be' D. men should ignore their animal needs E. men are different from animals as far as their reasoning abilities are

concerned.

Ans : E

Questions 17-21

Two or more essences out of a stock of five essences-- L, M, N, O, and P are used in making all perfumes by a manufacturer. He has learned that for a blend of essences to be agreeable it should comply with all the rules listed below.

A perfume containing L, should also contain the essence N, and the quantity of N should be twice as that of L. A perfume containing M, must also have O as one of its components and they should be in equal proportion. A single perfume should never contain N as well as O. O and P should not be used together. A perfume containing the essence P should contain P in such a proportion that the total amount of P present should be greater than the total amount of the other essence or essences used.

17.Among the following which is an agreeable formula for a perfume? A. One part L, one part P B. Two parts M, two parts L

9

Page 10: msd1

C. Three parts N, three parts L D. Four parts O, four parts M E. Five parts P, five parts M

Ans : D

18.Adding more amount of essence N will make which of the following perfumes agreeable?

A. One part L, one part N, five parts P B. Two parts M, two parts N, two parts P C. One part M, one part N, one part P D. Two parts M, one part N, four parts P E. Two parts N, one part O, three parts P

Ans : A

19.Among the following, the addition of which combination would make an unagreeable perfume containing two parts N and one part P agreeable?

(A) One part L (B) One part M (C) Two parts N (D) One part O (E) Two parts P

Ans : E

20.Among the following which combination cannot be used together in an agreeable perfume containing two or more essences?

A. L and M B. L and N C. L and P D. M and O E. P and N

Ans : A

21.Among the below mentioned formulas, which can be made agreeable by the eliminating some or all of one essence ?

A. One part L, one part M, one part N, four parts P B. One part L, two parts N, one part O, four parts P C. One part L, one part M, one part O, one part P D. Two parts L, two parts N, one part O, two parts P E. Two parts M, one part N, two parts O, three parts P

Ans : B

Questions 26 - 31

10

Page 11: msd1

Nine individuals - Z, Y, X, W, V, U, T, S and R - are the only candidates, who can serve on three committees-- A, B and C, and each candidate should serve on exactly one of the committees.

Committee A should consist of exactly one member more than committee B. It is possible that there are no members of committee C. Among Z, Y and X none can serve on committee A. Among W, V and U none can serve on committee G. Among T, S and R none can serve on committee C.

26. In case T and Z are the individuals serving on committee B, how many of the nine individuals should serve on committee C?

A. 3 B. 4 C. 5 D. 6 E. 7

Ans : B

27.Of the nine individuals, the largest number that can serve together on committee C is

A. 9 B. 8 C. 7 D. 6 E. 5

Ans : D

28. In case R is the only individual serving on committee B, which among the following should serve on committee A?

A. W and S B. V and U C. V and T D. U and S E. T and S

Ans : E

29. In case any of the nine individuals serves on committee C, which among the following should be the candidate to serve on committee A?

A. Z B. Y C. W D. T

11

Page 12: msd1

E. S

Ans : C

30. In case T, S and X are the only individuals serving on committee B, the total membership of committee C should be:

A. Z and Y B. Z and W C. Y and V D. Y and U E. X and V

Ans : A

31.Among the following combinations which could constitute the membership of committee C?

A. Y and T B. X and U C. Y, X and W D. W, V and U E. Z, X, U and R

Ans : B

Questions 32 - 34

(M, N, O and P are all different individuals)

I. M is the daughter of N. II. N is the son of O

III. O is the father of P.

32.Among the following statements, which is true ? A. O is the uncle of M. B. P and N are brothers C. M is the daughter of P. D. If B is the daughter of N, then M and B are sisters. E. If C is the granddaughter of O, then C and M are sisters.

Ans : D

33.Which among the following statements is contradictory to the above premises?

A. P is the father of M. B. O has three children. C. M has one brother.

12

Page 13: msd1

D. M is the granddaughter of O. E. Another party C, could be the mother of M.

Ans : A

34. If B is the son of N and B has one brother, D, then

I. M is the sister of D. II. D and N are brothers.

III. O is the grandfather of D.

A. I only B. II only C. III only D. I and III only E. I and II only

Ans : D

Questions 35 - 41

The only people to attend a conference were four ship captains and the first mates of three of those captains. The captains were L, M, N and O; the first mates were A, D and G. Each person in turn delivered a report to the assembly as follows:

Each of the first mates delivered their report exactly after his or her captain. The first captain to speak was M, and captain N spoke after him.

35.Among the following which is not an appropriate order of delivered reports?

A. M, A, N, G, O, L, D B. M, D, N, G, L, O, A C. M, N, A, L, D, O, G D. M, N, A, O, D, L, G E. M, N, G, D, O, L, A

Ans : E

36. In case L speaks after A, and A is the third of the first mates to speak, then among the following statements which would be untrue?

A. O spoke immediately after G. B. The order of the first four speakers was M, G, N, D. C. O's first mate was present. D. A was the fourth speaker after M. E. The captains spoke in the order M, N, O, L.

13

Page 14: msd1

Ans : D

37.Among the following statements which statement must be true? A. In case the second speaker was a captain, the seventh speaker

was a first mate. B. In case the second speaker was a first mate, the seventh speaker

was a captain. C. In case the third speaker was a first mate, the seventh speaker was

a captain. D. In case the third speaker was a captain, the seventh speaker was a

first mate. E. In case the seventh speaker was a first mate, the first and third

speakers were captains.

Ans : A

38. In case A spoke immediately after L and immediately before O, and O was not the last speaker, L spoke

A. second B. third C. fourth D. fifth E. sixth

Ans : C

39. In case G is M's first mate, D could be the person who spoke immediately A. prior to T B. prior to L C. prior to V D. after T E. after V

Ans : D

40. In case A is the third of the first mates to speak, and L is the captain whose first mate is not present, which among the following statements must be true?

A. A spoke sometime before L. B. D spoke sometime before O. C. L spoke sometime before O. D. O spoke sometime before L. E. O spoke sometime before N.

Ans : B

14

Page 15: msd1

41.Among the following statements, which would make M, D, N, G, L, O, A the only possible sequence of speakers?

A. D is M's first mate; G is N's first mate; A is O's first mate. B. D is M's first mate; G is N's first mate; A was the second to speak

after L. C. The order of the first four speakers was M, D, N, G. D. The order of the last three speakers was L, O, A. E. The order in which the captains spoke was M, N, L, O.

Ans : B

Questions 42 - 45

1. The microbe A causes the contagious disease A. 2. The first symptoms appear after a period of two days since the microbe A

enters the body. 3. The microbe A is found in some flies and bees. 4. A fly bit Jack on Monday, February 6. 5. Nick worked with Jack the next day, Tuesday, February 7.

There were no other possibilities of exposure to Disease A.

42. In case Jack showed symptoms of Disease A, which of the following statements would be true?

I. Jack contracted the Disease A from Nick. II. Jack first noticed symptoms of Disease A on February 8.

III. The fly that Jack was bitten by was not a carrier of the microbe A.

A. I only B. II only C. III only D. I and II only E. I and III only

Ans : B

43. In case Nick displayed symptoms of the disease A, which among the following would be true?

A. I only B. II only C. III only D. II and III only E. I, II and III

Ans : D

15

Page 16: msd1

44. In case Jack displayed symptoms of Disease A, which would be true?

I. Jack was also bitten by a fly on February 5. II. Jack was bitten by a mosquito which carried the microbe A.

III. Nick contracted Disease A from Jack.

1. I only 2. II only 3. III only 4. I and II only 5. II and III only

Ans : D

43. In case Nick displayed the symptoms of Disease A which would be true?

IV. Nick was bitten by a bee on February 6. V. Nick ate food which contained the microbe A. VI. Nick also worked with Jack on February 6.

1. I only 2. II only 3. III only 4. I and II only 5. I, II and III

Ans : A

Questions 46 - 47

The principal evil in today's society is selfishness. Everywhere we see people, who are concerned only with themselves. Personal advancement is the only motivating force in the world today. This does not mean that individuals are not willing to help one another; on the contrary, _________. But, these are only short-term occurrences which ultimately serve our long-term goal of personal gain.

46.To fill in the blank in the above passage, select one of the options from the below mentioned options:

1. we are always trying to undermine others’ endeavors. 2. my uncle Jeremy used to help me with my homework. 3. no one can be trusted, not even close friends. 4. our yearning for power prevents us from understanding our

existential purpose. 5. there are many occasions when we graciously offer our assistance.

16

Page 17: msd1

Ans : E

47.Which among the following options would most strongly contradict the author's attitude towards society?

1. The greatest strength of society is altruism. 2. The forces of good will ultimately triumph over evil. 3. Our short-term actions may ostensibly contradict our long-term

goals. 4. We must all learn the art of selfishness. 5. Morality is the bedrock of a growing community.

Ans : A

48. It can be proved by an example that our words are devoid of meaning as they cannot be distinguished from their opposites. People think that they are aware of the difference between the meanings of 'bald' and 'having hair' Let us suppose that an average twenty-one year old has X strands of hair on his or her head. We say that such a person is not bald but has hair. But surely one hair less would make no difference, and a person with X - 1 hairs on his or her head would be said to have hair. Suppose we go on like this, decreasing one hair at a time, the result would be the same. Then what difference would there be between someone who has one hair and someone who has none? We call them both bald. Thus, we cannot make a distinction between the terms 'bald' and 'having hair.'

Among the following statements, which statement best counters the argument above?

A. The word 'bald' can be translated into other languages. B. A word can have more than one meaning. C. A word such as 'monkey' can be applied to several animals that

differ in some respects. D. Words can lack precision without being meaningless. E. People cannot think clearly without using words.

Ans : D

49.Virus M helps in controlling the population of gnats; they manage to do this by killing the moth's larvae. Though the virus is always present in the larvae, it is only every six or seven years that the virus seriously decimates the numbers of larvae, greatly reducing the population of the gnats. Scientists are of the opinion that the gnats, usually latent, are activated only when the larvae experience biological stress.

17

Page 18: msd1

In case the above mentioned scientists are correct, it can be inferred that the decimation of gnat populations by the virus M could be most probably activated by the following conditions?

A. A shift by the gnats from drought areas to a normal area occupied by them

B. The resultant stress from defoliation of trees attacked by the gnats for the second consecutive year.

C. Attacks on the larvae by all kinds by parasitic wasps and flies. D. Starvation of the gnat larvae due to over population. E. Spraying of gnat infested areas with laboratory - raised Virus M.

Ans : D

50. In a particular code, the digits from 0 to 9 inclusive are each represented by a different letter of the alphabet, the letter always representing the same digit. In case the following sum

B O P B + S K B -------- C V B Q

holds true when it is expressed in digits, which of the following cannot be properly inferred:

A. B cannot be 0. B. B must be less than 5. C. Q must be even. D. O + S must be greater than 8. E. C must be greater than B by 1.

Ans : B

Questions 51 - 56

There are three on-off switches on a control panel A, B, and C. They have to be changed from an initial setting to a second setting according to the following conditions : In case only switch A is the switch on in the initial setting , then turn on switch B.

In case switches A and B are the only switches on in the initial setting, then turn on switch C. In case all the three switches are on initially setting, then turn off the switch C. For any other initial setting, turn on all switches that are off and turn off all switches, if any, that are on.

18

Page 19: msd1

51. In case in the initial setting is the switches A and B are on and the switch C is off, then what could be the second setting?

A. A on, B on, C on. B. A on, B off, C on. C. A on, B off, C off. D. A off, B on, C off. E. A off, B off, C on.

Ans : A

52. In case switch B is the only switch on in the initial setting, what must be the second setting?

A. A on, B on, C on. B. A on, B on, C off. C. A on, B off, C on. D. A off, B off, C on. E. A off, B off, C off.

Ans : C

53. In case all the three switches are on in the second setting, which among the following could have been the initial setting ?

A. A on, B on, C on. B. A on, B on, C off. C. A on, B off, C on. D. A on, B off, C off. E. A off, B on, C off

Ans : B

54. In case switch A is off in the second setting, which among the following could have been the initial setting ?

A. A on, B on, C on. B. A on, B on, C off. C. A on, B off, C on. D. A on, B off, C off. E. A off, B on, C off.

Ans : C

55. In case only switch B is on in the second setting, which among the following could have been the initial setting ?

A. A on, B on, C on. B. A on, B off, C on. C. A off, B on, C off. D. A off, B off, C on.

19

Page 20: msd1

E. A off, B off, C off.

Ans : B

56.Which among the following initial settings leads to a second setting, where only one switch is off ?

A. A on, B on, C off. B. A on, B off, C on. C. A off, B on, C on. D. A off, B on, C off. E. A off, B off, C off.

Ans : D

Questions 57 - 59

A bus has exactly six stops on its route. The bus first stops at stop one and then at stops two, three, four, five, and six respectively. After the bus leaves stop six, the bus turns and returns to stop one and repeats the cycle. The stops are at six building that are, in alphabetical order L, M, N, O, P, and Q.

P is the third stop. M is the sixth stop. The stop O is the stop immediately before Q. N is the stop immediately before L.

57. In case N is the fourth stop, which among the following must be the stop immediately before P ?

A. O B. Q C. N D. L E. M

Ans : B

58. In case L is the second stop, which among the following must be the stop immediately before M ?

A. N B. L C. P D. O E. Q

Ans : E

20

Page 21: msd1

59. In case a passenger gets on the bus at O, rides past one of the stops, and gets off at P, which of the following must be true ?

A. O is stop one. B. Q is stop three. C. P is stop four. D. N is stop five. E. L is stop six.

Ans : A

60.A survey recently conducted revealed that marriage is fattening. The survey found that on an average, women gained 23 pounds and men gained 18 pounds during 13 years of marriage. The answer to which among the following questions would be the most appropriate in evaluating the reasoning presented in the survey ?

A. Why is the time period of the survey 13 years, rather than 12 or 14 ?

B. Did any of the men surveyed gain less than 18 pounds during the period they were married ?

C. How much weight is gained or lost in 13 years by a single people of comparable age to those studied in the survey ?

D. When the survey was conducted were the women as active as the men?

E. Will the gains seen in the study be retained over the lifetimes of the surveyed persons?

Ans : C

61.Classical works occupy a unique and peculiar position. They are imperishable as cultural landmarks, but the views expressed in some of the works are often superseded by newer views. But the classics survive just like ancient castles, as imposing features of the landscape, yet unsuited for habitation unless remodeled.

The principal point put forth in the above passage is that classical works.

A. Demand our respect and admiration even if their ideas are no longer current.

B. Prove to be obstacles in the path of new ideas as they are unduly respected.

C. Should be preserved and treasured as scholarly opinion keeps changing and they will come into fashion again.

D. Lose their distinctive features as time passes. E. Are not given due consideration by the younger generation.

Ans : A

21

Page 22: msd1

62.Experts in the field of forestry are now questioning the policy of attempting to extinguish all forest fires instead of allowing them to run their course and die out on their own. The study of forest fires indicates that in the long run, forest fires may in fact, be beneficial to the ecology as a whole.

Among the following, which statement would logically follow from one of the ’observations’ referred to in the passage above ?

A. Most of the forest fires are causes due to natural reasons like lightning rather than as a result of human carelessness.

B. Every year the number of people dying as they attempt to fight forest fires is greater than the number of people who would have died or suffered injury if the fires were allowed to run their own course.

C. Accumulation of highly flammable dead brushwood and timber, which could lead to massive uncontrollable fires is prevented by frequent small forest fires.

D. The animal and plant life destroyed by forest fires seldom represents endangered species.

E. Studies indicate that forest fires regularly occurred in the times prior to human occupation of forest areas.

Ans : C

63.There is a theory that the dinosaurs inhabiting the earth were destroyed when a huge heavenly body hit the earth. The impact of the heavenly body when it struck the earth threw a mass of pulverized debris into the atmosphere. This dust created a fog, which blocked the sunlight depleting plant life. This resulted in the perishing of the plant-eating dinosaurs; the meat-eating dinosaurs, who preyed on the plant-eaters inturn starved to death.

Which among the following statements, would best support the above mentioned theory, if true?

A. Enough dust has been sent into the atmosphere by modern volcanic explosions to change the color of sunsets around the world for several years.

B. Craters formed by heavenly bodies are plentiful in several areas of the South west where many dinosaur fossils have also been found.

C. Studies conducted recently on the bone structure of dinosaurs show that they were very agile and not ponderous awkward animals as popularly believed.

D. The sedimentary rock strata of the earth formed from atmospheric dust at the time the dinosaurs disappeared contains minerals and

22

Page 23: msd1

other trace elements in proportions characteristic of the heavenly bodies.

E. Many meat-and plant-eating species of mammals who coexisted with the dinosaurs continue to exist today, although with a slightly modified appearance.

Ans : D

Questions 64 - 70

Five educational films A, B, C, D, & E are to be shown to a group of students. The films are to be shown in a particular order, which conforms to the following conditions:

A must be shown earlier than C. B must be shown earlier than D. E should be the fifth film shown.

64.Which among the following is an acceptable order for showing the educational films ?

A. A, C, B, D, E B. A, C, D, E, B C. B, D, C, A, E D. B, D, E, A, C E. E, B, C, A, D

Ans : D

65. In case C is shown earlier than E, which among the following will hold true ?

A. A is the first film shown. B. B is the second film shown. C. C is the third film shown. D. D is the fifth film shown. E. E is the fourth film shown.

Ans : D

66. In case D is to be shown earlier than A, then for which among the following is there exactly one position from first through fifth in which it can be scheduled to be shown ?

A. A B. B C. C D. D E. E

23

Page 24: msd1

Ans : C

67.Which among the following is a pair of films that CANNOT both be shown earlier than E ?

A. A and B B. A and D C. B and C D. B and D E. C and D

Ans : E

68. In case D and E are shown as far apart from each other as possible, which among the following would be true ?

A. A is shown earlier than B. B. B is shown earlier than C. C. C is shown earlier than E. D. D is shown earlier than A. E. E is shown earlier than B.

Ans : E

69. In case B, D and E are to be shown one after the other in the given order, the two positions from first to fifth in which A could possibly be shown are

A. first and second. B. first and fourth. C. second and third. D. third and fifth. E. fourth and fifth.

Ans : B

70. In case exactly one film is shown between A and C, and exactly one film is shown between B and D, which among the following will hold true ?

A. B is the film shown between A and C. B. C is the film shown between B and D. C. E is the film shown between A and C. D. D is the last film shown. E. E is the first film shown.

Ans : E

71.Authorities concerned with mass transport in metropolitan cities are struggling with deficits. Commuters complain about delays and breakdowns, cuts in service, and fares higher than they are accustomed to paying. For all these reasons and because the price of fuel is still not

24

Page 25: msd1

prohibitive, the number of commuters using public transportation has fallen, adding to the deficits.

Which among the following statements about the relationship between the number of commuters using public transportation and the price of fuel is best supported by the above passage?

A. With the rise in the price of fuel, there is a rise in the number of commuters using public transportation.

B. Even if the price of fuel rises, the number of commuters using public transportation will continue to decline.

C. The number of commuters using public transportation will rise, if the price of fuel rises to a prohibitive level.

D. Most of the commuters using public transportation do not use fuel, therefore fluctuations in the price of fuel is unlikely to affect the number of commuters using public transportation.

E. The price of fuel is always low enough to make private transportation cheaper than public transportation; therefore fluctuations in the price of fuel is unlikely to affect the number of commuters using public transportation.

Ans : C

72.Pharmaceutical firms are now engaged in the production of analogues of endorphins, peptides, which are thought to transmit messages among brain cells, which bring about pain relief. The pharmaceutical firms claim that the analogues, when injected into the bloodstream, will lead to effective and long-lasting pain relief by increasing the action of peptides already present in the brain.

The claims put forth by the pharmaceutical firms would be considerably weakened if it were true that

A. Compared to the other types of neurotransmitters present in the brain, endorphins remain active for a longer time period.

B. Certain peptides have been traced in parts besides the brain, like the alimentary canal and the skin.

C. It is easier and cheaper to produce analogues of peptides in the laboratory than the peptides themselves.

D. Analogues of peptides, which are found naturally in the body are often filtered from the blood before the blood circulates in the brain.

E. Endorphins interact chemically both with other naturally occurring peptides and with the brain's other neurotransmitters.

Ans : D

25

Page 26: msd1

73.Quite often it happens that a particular crisis or opportunity induces people to find a practical use for things, which originally had no serious purpose. To exemplify this principle, consider dolls and mannequins, programmed to move and built for the delight of the wealthy in the eighteenth century; these were the forerunners of the modern computer. Similarly, it is almost certain that pets were the first domesticated animals. Domestication of animals seems to have arisen as an amusement long before it had any practical application.

Which among the following, provides another example supporting the above mentioned principle, if true?

A. America was discovered as a by-product of the search for ginger, cloves, pepper, and cinnamon.

B. Children often imitate adults in their games. C. Till the commercial and scientific potential of the spyglass was

recognized and its power of magnification was improved it remained a mere source of diversion.

D. Some culture horses were used only for pleasure and not for work, even though in those cultures people had to labor hard in the absence of laboring animals.

E. People who made moving dolls and mannequins in the eighteenth century were also clockmakers.

Ans : C

74.Ever since the communications-entertainment firms have taken over publishing concerns, the management's new methods have increased the financial profits of commercial publishing. This is done at the price of narrowing the range of available books to the public and by catering to the vulgar tastes of the new buyers of books. There business has boomed ; but in the process the losers are the majority of authors or aspiring writers, and all discriminating readers.

In case the above statement is true, which among the following will hold true?

A. Profitable business ventures are relatively new in the publishing world.

B. The readership which commercial publishing caters to today is different from the readership served in the past.

C. The profits resulting from the methods introduced by communications-entertainment management will encourage literary writers to work against all the odds.

D. The limited range of books available to the public is directed toward a more discriminating audience.

26

Page 27: msd1

E. The public is not aware of the trend in the publishing industry, which tends to specialize in publishing blockbuster books.

Ans : B

75. It is popularly believed that teachers are more or less indifferent about the microcomputer technology. This assumption is false, or at least dated. A survey recently conducted indicated that 80 percent of the 7,000 surveyed teachers revealed a high level of interest in microcomputers.

Among the following statements which would most damage the above argument if proved to be true?

A. There was no attempt made in the survey to ascertain whether the surveyed teachers had any previous exposure to microcomputers.

B. Teachers interested in microcomputer technology were more likely to complete and return the questionnaires than others.

C. Irrespective of their subject area, their expertise and their teaching experience questionnaires were received by the teachers.

D. After the survey results were tabulated there have been many developments in the applications of microcomputer technology.

E. A company manufacturing and selling microcomputers conducted the survey.

Ans : B

Questions 76 - 79

Six scientists A, B, C, D, E, and F are to present a paper each at a one-day conference. Three of them will present their papers in the morning session before the lunch break whereas the other three will be presented in the afternoon session. The lectures have to be scheduled in such a way that they comply with the following restrictions:

B's should present his paper immediately before C's presentation; their presentations cannot be separated by the lunch break. D must be either the first or the last scientist to present his paper.

76. In case C is to be the fifth scientist to present his paper, then B must be A. first B. second C. third D. fourth E. sixth

Ans : D

27

Page 28: msd1

77.B could be placed for any of the following places in the order of presenters EXCEPT

A. first B. second C. third D. fourth E. fifth

Ans : C

78. In case F is to present his paper immediately after D presents his paper, C could be scheduled for which of the following places in the order of presenters?

A. First B. Second C. Third D. Fourth E. Fifth

Ans : E

79. In case F and E are the fifth and sixth presenters respectively then which of the following must be true?

A. A is first in the order of presenters. B. A is third in the order of presenters. C. A is fourth in the order of presenters. D. B is first in the order of presenters. E. C is fourth in the order of presenters.

Ans : C

Questions 80 - 82

In a small inn, one or more of the chefs have to perform duty during dinnereveryday. The chefs are Nicholas, Antonio, and Jeremy.

None of them can be assigned to dinner duty two or more days in a row.

80. In case Antonio and Jeremy share the dinner duty thrice over a five-day period, which among the following would be true?

A. Nicholas is on dinner duty alone on the first of the five days. B. Nicholas is on dinner duty alone on the second of the five days. C. Nicholas is on dinner duty alone on the third of the five days. D. Antonio and Jeremy share dinner duty on the second of the five

days.

28

Page 29: msd1

E. Antonio and Jeremy share dinner duty on the fourth of the five days.

Ans : C

81. In case Nicholas and Antonio share dinner duty on Monday of some week, and if Antonio and Jeremy share dinner duty on Thursday of the same week, which of the following would be true for that week?

A. Antonio is on dinner duty alone on Tuesday. B. Jeremy is on dinner duty alone on Wednesday. C. Nicholas and Jeremy share dinner duty on Wednesday. D. Nicholas is on duty alone on Tuesday, and Antonio is on breakfast

duty alone on Wednesday. E. Jeremy is on breakfast duty alone on Tuesday, and Nicholas is on

breakfast duty alone on Wednesday.

Ans : E

82.Which among the following could be true of some four-day period? A. On three of the four days, pairs of chefs were on dinner duty. B. On one of the four days, all three chefs were on dinner duty. C. Antonio drew twice as many dinner assignments as did Jeremy. D. Nicholas drew three times as many dinner assignments as did

Antonio. E. Both Nicholas and Antonio drew three times as many dinner

assignments as did Jeremy.

Ans : C

83.Which among the following statements is the most probable taking into consideration the source and the degree of seriousness with which they are uttered?

A. Examiner: "The exam will be taken under the honor system: the professors have the honor, and the students have the system."

B. Banker: "The low percentage of Jews in this bank has nothing to do with discrimination. Jews don't seem to be interested in banking."

C. Book cover: "This book is sure to become the last word in its field." D. Politician: "My opponent has an impeccable political and personal

record." E. Artist: "Einstein's theory of relativity falsifies life."

Ans : D

84.Tom: It is likely that Greece will be humiliated in the Cyprus affair. In case she fights Turkey, she will be defeated in battle; whereas if she doesn't

29

Page 30: msd1

fight, letting Turkey win, she will be humiliated, as she would then seem impotent.

Sarah: In that case Greece could always call the Soviet Union to her aid.

Sarah attacks Tom's argument by:

A. attacking Tom on a personal level. B. attacking Tom's method of reasoning C. siding with Greece D. advocating passive resistance E. citing an alternative which was overlooked by Tom.

Ans : E

85. In case Sue sits between Pete and Harry, then Harry sits between Sue and Mike. Harry won't be there unless Sue sits next to Mike. Hence, Sue will not sit between Pete and Harry.

Apart from the above mentioned statements, what additional premises are assumed by the author of this argument?

Mike sits next to Sue if no one sits between them. If Sue sits between Pete and Harry, then Sue sits between Harry and Pete.

If Harry isn't there, then he doesn't sit next to Mike.

A. I and II only B. I and III only C. II and III only D. I, II and III E. None of the above

Ans : B

Questions 86 - 89

An increasing number of people prefer to retain their own individuality and their own identity and consequently this has lead to a decline in the marriage rate.

86.Which among the following assumptions are used in the above premises? I. When a person is married, he or she loses his or her own identity

and is no longer accountable to himself or herself. II. Married persons do not find contentment as opposed to unmarried

people.

30

Page 31: msd1

III. There has been a steady increase in the divorce rate.

A. I only B. II only C. III only D. I and II only E. I, II, and III

Ans : A

87.Among the following statements, which would weaken the above argument?

A. The stability resulting from marriage offsets the negative aspects of the dual responsibility of husband and wife.

B. Most people are not mature enough to be married. C. Among most married couples the wife wants to have children. D. There are a differing set of values honored by men and women. E. It is advantageous to be single form a tax point of view.

Ans : A

88.Which among the following would strengthen the above argument? A. Very few people prefer to bring up a family. B. Emotionally divorce is not an easy procedure. C. 700 couples from 1000 surveyed couples complained that they

were losing their identity. D. Married people have to make a considerable effort to make the

marriage last. E. The financial complications arising from a divorce are becoming

decreasingly complicated.

Ans : C

89.Emma: Financially the private university is ruined. In case it raises its tuition fees, the number of paying students will come down and if they refrain from raising the fees the university will go bankrupt.

Richard: The pay of the teachers and professors could be cut. Richard responds to Emma’s argument by

A. drawing attention to an earlier overlooked alternative. B. attacking her method of reasoning. C. resigning himself to the demise of the private university. D. attacking Emma on a personal level. E. taking the side of the university as against that of the students.

31

Page 32: msd1

Ans : A

Questions 90 - 92

Mrs. Green wishes to renovate her cottage. She hires the services of a plumber, a carpenter, a painter, an electrician, and an interior decorator. The renovation is to completed in a period of one working week i.e. Monday to Friday. Every worker will be taking one complete day to do his job. Mrs. Green will allow just one person to work per day.

1. The painter can do his work only after the plumber and the carpenter have completed their jobs.

2. The interior decorator has to complete his job before that of the electrician. 3. The carpenter cannot work on Monday or Tuesday.

90. In case the painter works on Thursday, which among the following alternatives is possible?

A. The electrician works on Tuesday. B. The electrician works on Friday. C. The interior decorator does his work after the painter. D. The plumber and the painter work on consecutive days. E. Mrs. Green cannot fit all of the workers into schedule.

Ans : B

91. In case the painter works on Friday, which among the following statements must be untrue?

A. The carpenter may work on Wednesday. B. The carpenter and the electrician may work on consecutive days. C. In case the carpenter works on Thursday, the electrician has to

work on the previous day i.e. Wednesday. D. The plumber may work before the electrician does. E. The electrician may work on Tuesday.

Ans : C

92.Which arrangement among the following is possible? A. The electrician will work on Tuesday and the interior decorator on

Friday. B. The painter will work on Wednesday and the plumber on Thursday. C. The carpenter will work on Tuesday and the painter on Friday. D. The painter will work on Monday and the carpenter on Thursday. E. The carpenter will work on Wednesday and the plumber on

Thursday.

Ans : E

32

Page 33: msd1

93.According to the laws pertaining to the use of recyclable containers, beverages are sold only in reusable containers. The people who object to such laws point out that the collection of reusable bottles requires gasoline, and washing them requires water; but, it takes less water to wash these containers than it does to make throwaway bottles. The expenditure saved due to the saved water exceeds the cost of the gasoline used to transport empty bottles.

The above passage directly answers which of the below mentioned objections to recyclable beverage containers?

A. It is likely that consumers will continue to discard even the recyclable containers.

B. Consumers will find it an inconvenience to return bottles. C. The extra expense involved in recycling would raise the prices of

beverages. D. Recycling would be detrimental to the glass and plastic industries,

which produce containers. E. Recycling of containers only partially answers the problem of

disposal of solid waste.

Ans : C

94.A recently conducted study reveals that the prospects for success in later schooling for 3-5 old children were improved by a particular educational plan. From this it follows that the introduction of similar education plans for all 3-5 year old children will lead to an improvement in their chances for success in later schooling.

Which among the following would weaken the above argument if true?

A. Parents of preschool children in the United States get attracted to fads in the educational field, without having a clear idea of the type of early educational plans which would benefit their children.

B. The cognitive abilities of children are constantly changing between ages three and five

C. The people who conducted the research included quite a few children who had earlier been exposed to another educational enrichment program.

D. Many parents are of the view that formal training at an early age takes up the time, which the children could better spend exploring their worlds independently.

E. To establish such educational programs at a national level would require extraordinary public expense.

Ans : C

33

Page 34: msd1

Questions 95 - 97

Three girls Joan, Rita, and Kim and two boys Tim and Steve are the only dancers in a dance program, which consists of six numbers in this order: One a duet; two a duet; three a solo; four a duet; five a solo; and six a duet.

None of the dancers is in two consecutive numbers or in more than two numbers.

The first number in which Tim appears is the one that comes before the first number in which Kim appears. The second number in which Tim appears is one that comes after the second number in which Kim appears.

95.Which among the following is a complete and accurate list of those numbers that could be the last one in which Kim performs?

A. Three B. Four C. Five D. Three, Four E. Four, Five

Ans : E

96.Rita must perform only in duets if A. Kim is in number two B. Kim is in number five C. Tim is in number one D. Tim is in number two E. Tim is in number six

Ans : D

97. In case Steve is in number five, number four must consist of A. two women B. two men C. Tim and a woman D. Rita and a man E. Kim and a man

Ans : A

Refer to the test taking tips in the Logical Reasoning section. For the most part, they apply to the quantitative section as well.

1. READ THE QUESTION CAREFULLY! Make sure you understand exactly what you are being asked to do.

34

Page 35: msd1

2. Use your calculator! It will save you time and effort, and help you make accurate calculations.

3. Answers will often not work out to whole numbers or simple fractional values, as may have been the case on other standardized tests you have taken. This is another reason why calculators are recommended for this assessment.

4. Use the scratch paper provided. 5. If you are forced to guess because you cannot solve a problem or

run out of time, do not be too quick to select the "none of the above" option. This response is no more likely to be correct than any of the other alternatives. Instead, try to rule out response choices before you make your guess.

6. Review and practice working some problems in the topic areas listed below, especially if you have not worked with the concepts recently.

Q. 98 In general, the terms entrepreneur and SME’s (small and

medium enterprises) are used in conjunction with each other.  But

entrepreneurship is not just about owning a small business.  The very

basis of entrepreneurship, irrespective of the size of the organization,

lies in its ability to create new products/ services, and devising new

ways of implementing the existing or new products.

 

Which of the following best sums up the above passage?

 

(A)     The major characteristic of entrepreneurs is their ability to

introduce new products/services into the market.

(B)    Entrepreneurs are the main actors in economic growth.

(C)    An owner of a large business may be an entrepreneur.

(D)    Entrepreneurs do not own or operate small business.

35

Page 36: msd1

 

Q.99: During the summer this year, advertising expenditures on soft

drinks increased by 25%, while the consumption of soft drinks

increased by 30%.   Which of the following is irrelevant to explaining

the increase in consumption of soft drinks?

 

(A)     This summer, soft drink companies offered more volume

discounts than competing substitutes.

(B)    Soft drinks were available in more retail outlets this summer.

(C)    The advertisements of soft drinks were more catchy and

effective this summer.

(D)    The production of soft drink bottle openers doubled this year.

 

Instructions for Questions 100-101

 

A two-way road network exists between the following locations in a

city: A and B, A and C, C and E, E and G, E and H, G and D, and D and

F.  There is also a one-way road between locations D and B; the only

possible way of travel is from D to B.   None of these road routes

intersect each other except meeting at the nodal points in the

respective locations.  There are no other routes to or from the above

locations in the city.

36

Page 37: msd1

 

Q.100 Which of the following locations cannot be avoided while

traveling from F to H?

 

(A)     B, C

(B)    D, C

(C)    D, E

(D)    D, A

 

Q.101: What is the minimum number of locations one would have to

touch to reach E from F?

 

(A)     2

(B)    3

(C)    4

(D)    5

 

Q.102: The following sentences, when properly sequenced, form, a

coherent paragraph. Select the most logical order of the sentences.

37

Page 38: msd1

 

(i) Transmission and Distribution losses are very high in Indian State

Electricity Boards. (ii) Electricity rates have to be raised. (iii) State

Electricity Boards in India are making commercial losses. (iv) High

technical losses lead to loss of revenue and subsequent rise in

electricity prices.

 

(A)       (ii)-(i)-(iii)-(iv)

(B)      (iii)-(i)-(ii)-(iv)

(C)      (iii)-(i)-(iv)-(ii)

(D)      (i)-(ii)-(iii)-(iv)

 

Q.103: Seven MBA students are to be assigned projects, as part of

their curriculum. Three students (A, B and C) are engineering

graduates, two (D and E) are science graduates, and two (F and G)

are commerce graduates. The course instructor has offered them

three projects coded here as 1, 2 and 3.

 

No student can take part in more than one project.

There must be atleast one engineering graduate in each project.

B cannot be in the same project as G.

38

Page 39: msd1

C and F must work on the same project.

E must not work on project 3.

 

Since projects 1 and 2 are easier, the instructor has allowed only two

students to work on these projects.

 

Which of the following pairs CAN NOT work on the same project?

 

(A)       C and G

(B)      D and F

(C)      D and E

(D)      B and E

 

Topics covered in the Quantitative Part

You are expected to understand and use correctly a wide variety of basic mathematical concepts covered in most introductory level college mathematics courses.

This is an assessment of your basic quantitative skills, not your knowledge of advanced mathematics or economics. There is no calculus or matrix algebra, no econometrics or economic theory on the assessment. Although you might be tested on your understanding of linear regression, you will not be asked to develop any regression equations.

You should be prepared to solve problems involving such topics as:

39

Page 40: msd1

Decimals, fractions, and rounding procedures

Probability

Percentage and percentage change

Basic statistics, including

Ratios and proportions     -- Measures of central tendency (mean, median, and mode)

Inequalities     -- Weighted average

Linear equations     -- Standard deviation, variance, and other measures of dispersion

Non-linear functions (e.g., rate of change, growth rates)

Graphs and charts

Question 104

From 1992 to 1998, 2,139 fatalities resulted from job-related aircraft accidents. Of these fatalities, 853 were pilots. The other victims were "on the job" when they died and not employed as pilots.

According to the Federal Aviation Administration, the leading cause of aviation accidents is pilot error. During the period mentioned above, pilot error was cited as either a cause or a contributing factor in 77% of all aviation accidents and in 83% of fatal aviation accidents.

From these data it can be concluded that, during 1992 - 1998,

A) For aircraft accidents attributed to pilot error, the total number of non-fatal aircraft accidents was 6% less than the total number of fatal aircraft accidents

B) Slightly over 60% of fatalities resulting from job-related aircraft accidents occurred to non-pilots

C) The total percentage of aircraft accidents for which pilot error cannot be cited as a contributing factor (or cause) is approximately 17%

D) Less than 20% of all aircraft accidents involved no pilot fatalities

E) None of these

Which letter corresponds to the correct response? ______

Question 105

40

Page 41: msd1

A) For aircraft accidents attributed to pilot error, the total number of non-fatal aircraft accidents was 6% less than the total number of fatal aircraft accidents

B) Slightly over 60% of fatalities resulting from job-related aircraft accidents occurred to non-pilots

C) The total percentage of aircraft accidents for which pilot error cannot be cited as a contributing factor (or cause) is approximately 17%

D) Less than 20% of all aircraft accidents involved no pilot fatalities

E) None of these

The correct response is B. According to the passage, there were 2,139 fatalities resulting from job-related aircraft accidents during 1992-1998. Of these, 853, or approximately 39.88% (i.e., 853 ÷ 2,139), involved pilot fatalities. Therefore, as stated in response B, slightly over 60% of fatalities (i.e., 100% - 39.88%) occurred to non-pilots.

Response A is incorrect. The figure 6% is obtained by subtracting 77% from 83% in the question. But this subtraction is invalid because the two percentages are based on different total numbers. The 77% figure refers to all accidents (fatal and non-fatal), whereas the 83% refers to fatal accidents only.

Response C is also incorrect. The assertion in response C is that in approximately 17% of aircraft accidents, pilot error was not cited as a contributing factor. Option C does not limit the types of "aircraft accident". Therefore, it must refer to "all" aircraft accidents. The 83% figure in the passage refers to fatal accidents only. Therefore, 17% can only represent the percentage of fatal accidents not caused by pilot error. On the other hand, the passage states that pilot error was a cause or factor in 77% of all accidents. From this it follows that pilot error was not a factor in approximately 23% of all accidents.

The claim in response D that "less than 20% of all aircraft accidents involved no pilot fatalities" again involves subtracting 83% from 100% to get 17%. This misinterprets the 83% figure as applying only to those accidents in which there were pilot fatalities. The 83% figure actually refers to the percentage of all fatal accidents where pilot error was a contributing factor or cause.

Question 106

In July 1998, the following data were published for four categories of blue-collar workers in private industry in Huntsville, Alabama.

41

Page 42: msd1

OccupationNumber of workers

Mean hourly earnings

1. Precision production, craft, and repair occupations

7,235 $ 13.61

2. Machine operators, assemblers and inspectors

13,122 $ 14.08

3. Transportation an material moving occupations

1,391 $ 10.87

4. Handlers, equipment clearners, helpers, and laborers

2,910 $  9.05

Based on these data, the mean hourly earnings, rounded to the nearest cent, of a blue-collar worker who worked in private industry in Huntsville, Alabama, in 1998 was

A) $ 11.90

B) $ 11.91

C) $ 13.16

D) $ 13.17

E) None of these

Which letter corresponds to the correct response? ______

Question 107

A) For aircraft accidents attributed to pilot error, the total number of non-fatal aircraft accidents was 6% less than the total number of fatal aircraft accidents

B) Slightly over 60% of fatalities resulting from job-related aircraft accidents occurred to non-pilots

C) The total percentage of aircraft accidents for which pilot error cannot be cited as a contributing factor (or cause) is approximately 17%

42

Page 43: msd1

D) Less than 20% of all aircraft accidents involved no pilot fatalities

E) None of these

The correct response is B. According to the passage, there were 2,139 fatalities resulting from job-related aircraft accidents during 1992-1998. Of these, 853, or approximately 39.88% (i.e., 853 ÷ 2,139), involved pilot fatalities. Therefore, as stated in response B, slightly over 60% of fatalities (i.e., 100% - 39.88%) occurred to non-pilots.

Response A is incorrect. The figure 6% is obtained by subtracting 77% from 83% in the question. But this subtraction is invalid because the two percentages are based on different total numbers. The 77% figure refers to all accidents (fatal and non-fatal), whereas the 83% refers to fatal accidents only.

Response C is also incorrect. The assertion in response C is that in approximately 17% of aircraft accidents, pilot error was not cited as a contributing factor. Option C does not limit the types of "aircraft accident". Therefore, it must refer to "all" aircraft accidents. The 83% figure in the passage refers to fatal accidents only. Therefore, 17% can only represent the percentage of fatal accidents not caused by pilot error. On the other hand, the passage states that pilot error was a cause or factor in 77% of all accidents. From this it follows that pilot error was not a factor in approximately 23% of all accidents.

The claim in response D that "less than 20% of all aircraft accidents involved no pilot fatalities" again involves subtracting 83% from 100% to get 17%. This misinterprets the 83% figure as applying only to those accidents in which there were pilot fatalities. The 83% figure actually refers to the percentage of all fatal accidents where pilot error was a contributing factor or cause.

Question 108

Game testers play games for a living. They play each and every part of a game in order to identify possible errors in video game software, so the errors can be fixed before the software is shipped to stores. In a recent year, testers found that among all the games that they tested, 12% contained technical errors (for example, game characters who walked through walls, and sound or artwork that disappeared unpredictably). Nine percent (9%) contained errors in the game-play itself (e.g., portions that were too difficult, too easy, or too confusing for members of the intended audience.) The testers found that these two types of error were independent of one another.

From this information it follows that, if a game is selected at random from all the games that were tested, the probability that it will be found to contain both types of error

43

Page 44: msd1

A) Cannot be estimated

B) Is between 0.01 and 0.02

C) Is equal to 0.12

D) Is between 0.19 and 0.20

E) None of these

Which letter corresponds to the correct response? ______

) Cannot be estimated

B) Is between 0.01 and 0.02

C) Is equal to 0.12

D) Is between 0.19 and 0.20

E) None of these

The correct response is B. The error rates of 12% (or 0.12) and 9% (or 0.09) apply to "all the games that were tested" and so they can be used to solve the problem as posed. The probability of the joint occurrence of two independent events (in this case, a technical error and a game-play error) is equal to the product of the individual probabilities of occurrence. In this case, the joint probability equals 0.12 times 0.09 or 0.0108. Therefore, the probability value for the conjunction is between 0.01 and 0.02, which is within the range given by response B.

Response A is incorrect in claiming that the probability cannot be estimated. The estimation has just been demonstrated.

From the information given, the error in response C probably involves assuming that all pieces of software with a game-play error (12%) also contain a technical error. This would contradict the information given, which is that the two types of error occur independently.

The task in question 3 is to determine the probability that both types of error will be present, in a game selected at random from the defined set of all games that were tested. Response D results from erroneously treating the task as one of calculating the probability that either one event or the other, but not both, will occur. Accordingly, the value given in response D is calculated by adding the individual probabilities of occurrence for each of the two events and then subtracting the probability of their joint

44

Page 45: msd1

occurrence, thus: 0.12 + 0.09 - 0.0108 = 0.1992. This, however, is not the result the question asks for.

LECTURE 5-10

Quantitative Section : Quantitative Comparison

Directions:

In this section you will be given two quantities, one in column A and one in column B. You are to determine a relationship between the two quantities and mark.

A. If the quantity in column A is greater than the quantity in column B. B. If the quantity in column B is greater than the quantity in column A. C. If the quantities are equal. D. If the comparison cannot be determined from the information that is given.

1. Quantity A: (-6)4 Quantity B: (-6)5

A. if the quantity A is greater; B. if the quantity B is greater; C. if the two quantities are equal; D. if the relationship cannot be determined from the information given.

Ans : A

2. Quantity A: Time to travel 95 miles at 50 miles per hour Quantity B: Time to travel 125 miles at 60 miles per hour

A. Quantity A is greater B. Quantity A equals Quantity B C. Quantity B is greater D. Relationship Indeterminate

Ans : C

3. Quantity A: (9/13)2 Quantity B: (9/13)1/2

A. Quantity A equals Quantity B B. Relationship Indeterminate C. Quantity B is greater D. Quantity A is greater

Ans : C

45

Page 46: msd1

4. Quantity A: 4 / 100 Quantity B: 0.012 / 3

A. Quantity B is greater B. Quantity A equals Quantity B C. Quantity A is greater D. Relationship Indeterminate

Ans : C

5. x = 2y + 3 y = -2

Quantity A: x Quantity B: -1

A. if the quantity in Column A is greater; B. if the quantity in Column B is greater; C. if the two quantities are equal; D. if the relationship cannot be determined from the information given

Ans : C

6. x + 2y > 8

Quantity A: 2x + 4y Quantity B: 20

A. if the quantity in Column A is greater; B. if the quantity in Column B is greater; C. if the two quantities are equal; D. if the relationship cannot be determined from the information given.

Ans : D

7. Quantity A: The number of months in 7 years Quantity B: The number of days in 12 weeks

A. if the quantity in Column A is greater; B. if the quantity in Column B is greater; C. if the two quantities are equal; D. if the relationship cannot be determined from the information given

Ans : C

46

Page 47: msd1

8. Quantity A: 1-1/27 Quantity B: 8/9 + 1/81

A. if the quantity in is greater; B. if the quantity in is greater; C. if the two quantities are equal; D. if the relationship cannot be determined from the information given.

Ans : A

9. r/>s/>0/>

Quantity A: rs/r Quantity B: rs/s

A. if the quantity A is greater; B. if the quantity B is greater; C. if the two quantities are equal; D. if the relationship cannot be determined from the information given.

Ans : B

10.Quantity A: 0.83 Quantity B: 0.81/3

A. Quantity B is greater B. Relationship Indeterminate C. Quantity A is greater D. Quantity A equals Quantity B

Ans : A

11. t is a positive integer. 4/7 = t/s

Quantity A: s Quantity B:7

A. if the quantity in Column A is greater; B. if the quantity in Column B is greater; C. if the two quantities are equal; D. if the relationship cannot be determined from the information given

47

Page 48: msd1

Ans : D

12.Quantity A: (0.82)2(0.82)3 Quantity B:(0.82)6

A. if the quantity in Column A is greater; B. if the quantity in Column B is greater; C. if the two quantities are equal; D. if the relationship cannot be determined from the information given.

Ans : A

13.For all real numbers a, let a* = 1 - a.

Quantity A: ((-1)*)* Quantity B: 2*

A. if the quantity in Column A is greater; B. if the quantity in Column B is greater; C. if the two quantities are equal; D. if the relationship cannot be determined from the information given.

Ans : C

14.Quantity A: (x - 1)(x)(x + 1) Quantity B:(x)(x)(x)

A. if the quantity in Column A is greater; B. if the quantity in Column B is greater; C. if the two quantities are equal; D. if the relationship cannot be determined from the information given.

Ans : D

15.Quantity A: (3 x 4 x 17) / (121 x 100) Quantity B: (4 x 5 x 19) / (1000 x 121)

A. Quantity A is greater B. Quantity A equals Quantity B C. Relationship Indeterminate D. Quantity B is greater

A

48

Page 49: msd1

16.Consider a triangle PQR.

Quantity A: length of PQ + length of QR Quantity B: length of PR

A. Quantity A is greater B. Quantity B is greater C. Relationship Indeterminate D. Quantity A equals Quantity B

Ans : A

17.Quantity A: (27 - 13) (296 + 534) Quantity B: (27 + 13) (534 + 296)

A. Quantity B is greater B. Quantity A equals Quantity B C. Relationship Indeterminate D. Quantity A is greater

Ans : D

18.Quantity A: A = 1.1 Quantity B: 12.11/2

A. Relationship Indeterminate B. Quantity B is greater C. Quantity A equals Quantity B D. Quantity A is greater

Ans : B

19.100 < y < 200 and 100 < z < 210

Quantity A: y Quantity B: z

A. Quantity A is greater B. Quantity A equals Quantity B C. Quantity B is greater D. Relationship Indeterminate

49

Page 50: msd1

Ans : D

20.y2 + z2 = 34 and yz = 15

Quantity A: y2 + 2yz + z2 Quantity B: (y + z)2

A. Quantity B is greater B. Relationship Indeterminate C. Quantity A is greater D. Quantity A equals Quantity B

Ans : D

21.Consider a rectangle. The length of its shorter side is 8, and the length of its diagonal is 16.

Quantity A: 30o Quantity B: measure of angle formed by diagonal and shorter side

A. Relationship Indeterminate B. Quantity A equals Quantity B C. Quantity A is greater D. Quantity B is greater

Ans : D

22.Quantity A: (y + 5)2 Quantity B: (y - 5)2

A. Quantity B is greater B. Relationship Indeterminate C. Quantity A equals Quantity B D. Quantity A is greater

Ans : B

23.Quantity A: (1/25)1/2 + (1/144)1/2 Quantity B: [(1/25) + (1/144)]1/2

A. Relationship Indeterminate B. Quantity A is greater C. Quantity B is greater D. Quantity A equals Quantity B

50

Page 51: msd1

Ans : A

24.y2 + z2 = 34 and yz = 15

Quantity A: y2 + 2yz + z2 Quantity B: (y + z)2

A. Quantity A is greater B. Relationship Indeterminate C. Quantity A equals Quantity B D. Quantity B is greater

Ans : C

25.100 < y < 200 and 100 < z < 210

Quantity A: y Quantity B: z

A. Quantity A is greater B. Quantity A equals Quantity B C. Quantity B is greater D. Relationship Indeterminate

Ans : D

26.Quantity A: (y + 5)2 Quantity B: (y - 5)2

A. Quantity A equals Quantity B B. Quantity A is greater C. Relationship Indeterminate D. Quantity B is greater

Ans : C

27.Consider a rectangle. The length of its shorter side is 8, and the length of its diagonal is 16.

Quantity A: 30o Quantity B: measure of angle formed by diagonal and shorter side

A. Quantity A is greater

51

Page 52: msd1

B. Quantity A equals Quantity B C. Quantity B is greater D. Relationship Indeterminate

Ans : C

28.The sum of three consecutive even numbers is 18.

Quantity A: Their average Quantity B: 6

A. Relationship Indeterminate B. Quantity A is greater C. Quantity A equals Quantity B D. Quantity B is greater

Ans : C

29.x - y > 10

Quantity A: y - x Quantity B: 12

A. Quantity B is greater B. Quantity A is greater C. Quantity A equals Quantity B D. Relationship Indeterminate

Ans : A

30.x = 0, y > 0

Quantity A: xy Quantity B: yx

A. Quantity A equals Quantity B B. Quantity A is greater C. Quantity B is greater D. Relationship Indeterminate

Ans : C

52

Page 53: msd1

31.

Diagram is illustrative and is not drawn to scale.

Quantity A: Measure of angle 3 - Measure of angle 2 Quantity B: Measure of angle 5 - Measure of angle 6

A. Relationship Indeterminate B. Quantity A equals Quantity B C. Quantity B is greater D. Quantity A is greater

Ans : B

32.Quantity A: 29 Quantity B: 92

A. Quantity B is greater B. Quantity A is greater C. Relationship Indeterminate D. Quantity A equals Quantity B

Ans : B

33.0 < -x < 10 11 < -y < 20

Quantity A: x Quantity B: y

A. Relationship Indeterminate B. Quantity A equals Quantity B C. Quantity B is greater D. Quantity A is greater

Ans : D

53

Page 54: msd1

34.

Diagram is illustrative and is not drawn to scale. Given angles 1 and 2 are equal, length of side AB = x, length of side BC = y, length of side AC = z.

Quantity A: x + y Quantity B: y + z

A. Quantity A is greater B. Quantity A equals Quantity B C. Quantity B is greater D. Relationship Indeterminate

Ans : B

35.

Diagram is illustrative and is not drawn to scale. In triangle ABC, AB = AC and measure of angle 1 = 100o.

Quantity A: Measure of angle 2 + Measure of angle 3 Quantity B: 90o

A. Quantity B is greater B. Relationship Indeterminate C. Quantity A is greater D. Quantity A equals Quantity B

54

Page 55: msd1

Ans : A

36.x and y are both positive and x/y > 5

Quantity A: 0.2x Quantity B: y

A. Quantity A is greater B. Quantity B is greater C. Relationship Indeterminate D. Quantity A equals Quantity B

Ans : A

37.

Diagram is illustrative and is not drawn to scale. Given AB = AC and angle BAC = 60o

Quantity A: Length of side AB Quantity B: Length of side BC

A. Quantity A equals Quantity B B. Quantity B is greater C. Relationship Indeterminate D. Quantity A is greater

Ans : A

38.y2 = 36

Quantity A: y Quantity B: 6

A. Relationship Indeterminate B. Quantity A is greater C. Quantity B is greater D. Quantity A equals Quantity B

55

Page 56: msd1

Ans : A

39.

Diagram is illustrative and is not drawn to scale.

Quantity A: Measure of angle 1 + Measure of angle 2 + Measure of angle 4 Quantity B: 180o

A. Relationship Indeterminate B. Quantity A is greater C. Quantity B is greater D. Quantity A equals Quantity B

Ans : D

40.

Diagram is illustrative and is not drawn to scale. In triangle ABC, angle A = 60o and AB = AC.

Quantity A: Measure of angle 1 + Measure of angle 2 Quantity B: 120o

A. Relationship Indeterminate B. Quantity A is greater C. Quantity A equals Quantity B D. Quantity B is greater

Ans : C

56

Page 57: msd1

41.

Diagram is illustrative and is not drawn to scale.

Quantity A: Measure of angle 2 + Measure of angle 3 Quantity B: 180o

A. Quantity B is greater B. Quantity A is greater C. Quantity A equals Quantity B D. Relationship Indeterminate

Ans : C

42.

Diagram is illustrative and is not drawn to scale. AB is the diameter of the circle.

Quantity A: Measure of angle 1 Quantity B: Measure of angle 2

A. Relationship Indeterminate B. Quantity A is greater C. Quantity B is greater D. Quantity A equals Quantity B

Ans : D

57

Page 58: msd1

43.

Diagram is illustrative and is not drawn to scale.

Quantity A: Measure of angle 1 + Measure of angle 3 Quantity B: Measure of angle 2 + Measure of angle 4

A. Relationship Indeterminate B. Quantity A is greater C. Quantity A equals Quantity B D. Quantity B is greater

Ans : A

44.

Diagram is illustrative and is not drawn to scale. In triangle ABC, AB = AC and measure of angle 1 = 100o.

Quantity A: Measure of angle 2 + Measure of angle 3 Quantity B: 90o

A. Quantity B is greater B. Quantity A equals Quantity B C. Relationship Indeterminate D. Quantity A is greater

Ans : A

58

Page 59: msd1

45.

Diagram is illustrative and is not drawn to scale. Given angles 1 and 2 are equal, length of side AB = x, length of side BC = y, length of side AC = z.

Quantity A: x + y Quantity B: y + z

A. Quantity B is greater B. Quantity A equals Quantity B C. Quantity A is greater D. Relationship Indeterminate

Ans : B

46.x and y are both positive and x/y > 5

Quantity A: 0.2x Quantity B: y

A. Quantity B is greater B. Relationship Indeterminate C. Quantity A equals Quantity B D. Quantity A is greater

Ans : D

47.yz < 0

Quantity A: (y - z)2 Quantity B: y2 + z2

A. Quantity A is greater B. Quantity B is greater C. Quantity A equals Quantity B D. Relationship Indeterminate

59

Page 60: msd1

Ans : A

48.For any positive integer n, n! is the product of all positive integers less than or equal to n.

Quantity A: 20! / 17! Quantity B: 80! / 78!

A. Quantity A is greater B. Quantity B is greater C. Quantity A equals Quantity B D. Relationship Indeterminate

Ans : A

49.2 < z < 4

Quantity A: π2z3 Quantity B: π3z2

A. Quantity A is greater B. Quantity B is greater C. Quantity A equals Quantity B D. Relationship Indeterminate

Ans : D

50.Amy, Beth and Charlie divided a pizza amongst themselves. Amy took 30% of the pizza and ate (3/4) of what she took. Beth took 20% of the pizza. Charlie ate (2/5) of what he took.

Quantity A: The amount Amy ate Quantity B: The amount Charlie ate

A. Quantity A is greater B. Quantity B is greater C. Quantity A equals Quantity B D. Relationship Indeterminate

Ans : A

51.p > 0 > q

Quantity A: p + q Quantity B: pq

60

Page 61: msd1

A. The quantity in Column A is greater. B. The quantity in Column B is greater. C. The quantities are equal. D. The relationship cannot be determined from the information given.

Ans : D

52.Quantity A: The average (arithmetic mean) of x and y Quantity B: The average (arithmetic mean) of x - 1 and y + 1

A. The quantity in Column A is greater. B. The quantity in Column B is greater. C. The quantities are equal. D. The relationship cannot be determined from the information given.

Ans : C

53.The integer (x - 1) is a prime number between 40 and 50.

Quantity A: The sum of all different prime factors of x Quantity B: 14

A. The quantity in Column A is greater. B. The quantity in Column B is greater. C. The quantities are equal. D. The relationship cannot be determined from the information given.

Ans : B

54.A < C B > D > 0

Quantity A: A - B Quantity B: C - D

A. The quantity in Column A is greater. B. The quantity in Column B is greater. C. The quantities are equal. D. The relationship cannot be determined from the information given.

Ans : B

55. In a particular jellybean jar, the number of red jellybeans exceeds the number of white ones by a ratio of 3:2. If two red jellybeans were removed, the ratio of red to white jellybeans would be 1:1.

61

Page 62: msd1

Quantity A: The number of white jellybeans in the jar Quantity B: 4

A. The quantity in Column A is greater. B. The quantity in Column B is greater. C. The quantities are equal. D. The relationship cannot be determined from the information given.

Ans : C

62

Page 63: msd1

LECTURE 5-10

Quantitative Section : Quantitative Ability

Directions:In this section you will be given two quantities, one in column A and one in column B. You are to determine a relationship between the two quantities and mark.

A. If the quantity in column A is greater than the quantity in column B. B. If the quantity in column B is greater than the quantity in column A. C. If the quantities are equal. D. If the comparison cannot be determined from the information that is given.

1. A rectangle is 14 cm long and 10 cm wide. If the length is reduced by x cms and its width is increased also by x cms so as to make it a square then its area changes by :

A. 4 B. 144 C. 12 D. 2 E. None of the above.

Ans : A

2. A motorcycle stunts man belonging to a fair, rides over the vertical walls of a circular well at an average speed of 54 kph for 5 minutes. If the radius of the well is 5 meters then the distance traveled is:

A. 2.5 kms B. 3.5 kms C. 4.5 kms D. 5.5 kms E. None of the above

Ans : C

3. If 1 cm on a map corresponds to an actual distance of 40 kms. And the distance on the map between Bombay and Calcutta is 37.5 cms., the actual distance between them is :

A. 375 kms B. 3750 kms C. 1500 kms D. 1375 kms E. None of the above

Ans : C

63

Page 64: msd1

4. A box contains 90 mts each of 100 gms and 100 bolts each of 150 gms. If the entire box weighs 35.5 kg., then the weight of the empty box is :

A. 10 kg B. 10.5 kg C. 11 kg D. 11.5 kg E. None of the above

Ans : D

5. If the radius of a circle is increased by 20% then the area is increased by : A. 44% B. 120% C. 144% D. 40% E. None of the above

Ans : A

6. Tom, Dick and Harry went for lunch to a restaurant. Tom had $100 with him, Dick had $60 and Harry had $409. They got a bill for $104 and decided to give a tip of $16. They further decided to share the total expenses in the ratio of the amounts of money each carried. The amount of money which Tom paid more than what Harry paid is

A. 120 B. 200 C. 60 D. 24 E. 36

Ans : E

7. A plot of land is in the shape of a trapezium whose dimensions are given in the figure below :

Hence the perimeter of the field is A. 50 m B. 64 m C. 72 m D. 84 m E. None of the above

64

Page 65: msd1

Ans : c

8. Four concentric ( having the same center ) circles with radii, x, 2x, 3x and 4x are drawn to form two rings A and B as shown in the figure.

Ratio of the area of inner ring A to the area of outer ring B is A. 1 : 2 B. 1 : 4 C. 2 : 3 D. 3 : 7 E. None of the above

Ans : D

9. If 3/p = 6 and 3/q = 15 then p - q = ? A. 1/3 B. 2/5 C. 3/10 D. 5/6 E. None of the above

Ans : C

10.A father is three times as old as his son. After fifteen years the father will be twice as old as his son's age at that time. Hence the father's present age is

A. 36 B. 42 C. 45 D. 48 E. None of the above

Ans : C

11. (1/4)3 + (3/4)3 + 3(1/4)(3/4)(1/4 + 3/4) =? A. 1/64 B. 27/64 C. 49/64 D. 0

65

Page 66: msd1

E. 1

Ans : E

12. If the area of two circles are in the ratio 169 : 196 then the ratio of their radii is

A. 10 : 11 B. 11 : 12 C. 12 : 13 D. 13 : 14 E. None of the above

Ans : D

13.A semi-circle is surmounted on the side of a square. The ratio of the area of the semi-circle to the area of the square is

A. 1 : 2 B. 2 : p C. p : 8 D. 8 : p E. None of the above

Ans : C

14.Which of the following is the greatest ? A. 40% of 30 B. 3/5 of 25 C. 6.5% of 200 D. Five more than the square of 3 E. 1/2-4

Ans : E

15.Two identical taps fill 2/5 of a tank in 20 minutes. When one of the taps goes dry in how many minutes will the remaining one tap fill the rest of the tank ?

A. 5 minutes B. 10 minutes

66

Page 67: msd1

C. 15 minutes D. 20 minutes E. None of the above

Ans : C

16. If the value of XYZ Company stock drops from $25 per share to $21 per share, what is the percent of the decrease?

A. 4 B. 8 C. 12 D. 16 E. 20

Ans : D

17. If a building b feet high casts a shadow f feet long, then, at the same time of day, a tree t feet high will cast a shadow how many feet long?

A. ft/b B. fb/t C. b/ft D. tb/f E. t/fb

Ans : A

18. If x, y, and z are consecutive negative integers, and if x > y > z, which of the following must be a positive odd integer?

A. xyz B. (x - y) (y - z) C. x - yz D. x(y + z) E. x + y + z

Ans : B

19.At a certain ice cream parlor, customers can choose among five different ice cream flavors and can choose either a sugar cone or a waffle cone. Considering both ice cream flavor and cone type, how many distinct triple-scoop cones with three different ice cream flavors are available?

A. 12 B. 16 C. 20 D. 24 E. 30

67

Page 68: msd1

Ans : C

20.What is the greatest value of a positive integer n such that 3n is a factor of 1815?

A. 15 B. 18 C. 30 D. 33 E. 45

Ans : C

21. If .2t = 2.2 - .6s and .5s = .2t + 1.1, then s = A. 1 B. 3 C. 10 D. 11 E. 30

Ans : B

22.Five years ago, Beth's age was three times that of Amy. Ten years ago, Beth's age was one half that of Chelsea. If C repre- sents Chelsea's current age, which of the following represents Amy's current age?

A. c/6 + 5 B. 2c C. (c-10)/3 D. 3c-5 E. 5c/3 - 10

Ans : A

23.A portion of $7200 is invested at a 4% annual return, while the remainder is invested at a 5% annual return. If the annual income from both portions is the same, what is the total income from the two investments?

A. $160 B. $320 C. $400 D. $720 E. $1,600

Ans : B

24.An empty swimming pool can be filled to capacity through an inlet pipe in 3 hours, and it can be completely drained by a drainpipe in 6 hours. If both

68

Page 69: msd1

pipes are fully open at the same time, in how many hours will the empty pool be filled to capacity?

A. 4 B. 4.5 C. 5 D. 5.5 E. 6

Ans : E

25. If r = (3p + q)/2 and s = p - q, for which of the following values of p would r2 = s2?

A. 1q/5 B. 10 - 3q/2 C. q - 1 D. 3q E. 9q/2 - 9

Ans : A

26.At 10 a.m. two trains started traveling toward each other from stations 287 miles apart. They passed each other at 1:30 p.m. the same day. If the average speed of the faster train exceeded the average speed of the slower train by 6 miles per hour, which of the following represents the speed of the faster train, in miles per hour?

A. 38 B. 40 C. 44 D. 48 E. 50

Ans : C

27.On the xy-coordinate plane, points A and B both lie on the circumference of a circle whose center is O, and the length of AB equals the circle's diameter. If the (x,y) coordinates of O are (2,1) and the (x,y) coordinates of B are (4,6), what are the (x,y) coordinates of A?

A. (3, 3/2) B. (1, 2/2) C. (0, -4) D. (2/2, 1) E. (-1, -2/2)

Ans : C

69

Page 70: msd1

28. If a rectangle's length and width are both doubled, by what percent is the rectangle's area increased?

A. 50 B. 100 C. 200 D. 300 E. 400

Ans : D

29.A rectangular tank 10" by 8" by 4" is filled with water. If all of the water is to be transferred to cube-shaped tanks, each one 3 inches on a side, how many of these smaller tanks are needed?

A. 9 B. 12 C. 16 D. 21 E. 39

Ans : B

30.Point Q lies at the center of the square base (ABCD) of the pyramid pictured above. The pyramid's height (PQ) measures exactly one half the length of each edge of its base, and point E lies exactly halfway between C and D along one edge of the base. What is the ratio of the surface area of any of the pyramid's four triangular faces to the surface area of the shaded triangle?

A. 3 :√2 B. √5:1 C. 4√3:3 D. 2√2:1 E. 8:√5

Ans : D

Quantitative Section : Data Interpretation

Questions31 -3 5 refers to the following table:

PROFILE OF CONGRESS IN YEAR X (total membership: 535)

House ofRepresentatives

  Senate

  Party  

70

Page 71: msd1

292 Democratic 62143 Republican 38

435 TOTAL100

  Sex  

418 Male 100

17 Female 0

  Age  

27 Youngest 34

77 Oldest 80

48Average

(arithmetic mean)54

  Religion  

255 Protestant 69

107 Catholic 12

18 Jewish 5

4 Mormon 3

51 Other11

House ofRepresentatives

  Senate

  Profession  215 Lawyer 63

81Business Executive

or Banker15

45 Educator 614 Farmer or Rancher 622 Career Government 024 Official 4

2Journalist or

Communications0

1 Executive 10 Physician 2

71

Page 72: msd1

6Veterinarian

Geologist0

25Worker or Skilled

TradespersonOther

3

  Ethnic Group  

17 Black American 1

2 Asian American 3

4 Hispanic American0

31. In the Senate, if 25 male members were replaced by 25 female members, the ratio of male members to female members would be

A. 4 to 1 B. 3 to 1 C. 3 to 2 D. 2 to 1 E. 1 to 1

Ans : B

32.Approximately what percent of the members of Congress are lawyers? A. 63% B. 58% C. 56% D. 52% E. 49%

Ans : D

33. If 5 senators are Catholic Democrats, how many senators are neither Catholic nor Democratic?

A. 79 B. 74 C. 69 D. 31 E. 21

Ans : D

34. If all lawyers and all women in the House of Representatives vote for the passage of a bill, how many more votes will be needed for a majority?

A. 435 B. 220

72

Page 73: msd1

C. 3 D. 0 E. It cannot be determined from the information given.

Ans : E

35.Which of the following can be inferred from the information given in the chart?

I. More than 80 percent of the men in Congress are members of the House of Representatives.

II. The percent of members who are categorized as farmers or ranchers is greater for the House of Representatives than for the Senate.

III. The median age in the Senate is 57.

A. I only B. II only C. III only D. I and II E. I and III

Ans : A

Section : Quantitative Ability

 

Q.36: A text book for children is meant to have 216 sq. cms. of actual

printed matter in each page.  Also, the top and bottom margins are 3

cms. each and the left and right margins are 2 cms. each.  The most

economical height and width of each page will be respectively

 

 (A)   16 cms. and 12 cms.

 (B)   20 cms. and 14 cms.

(C)     24 cms. and 16 cms.

73

Page 74: msd1

(D)     18 cms. and 12 cms.

 

Q.37: The probability of a number being divisible by 3, not divisible by

5 and divisible by either 4 or 6 is

 

(A)     1/6

(B)    2/15

(C)    1/30

(D)    5/6

 

Q.38.  Let PQR be a right-angled triangle, right-angled at R, and let RS

be the perpendicular from R to PQ.  Let PQ=a, QR=b, RP=c, RS=d and

PS=e.  Which one of the following is not always true?

 

(A)     ad=bc

(B)    b2 + c2 = 2c2 + ½ a2

(C)    1/d2 =1/b2 + 1/c2

(D)    d2 = e(a-e) 

Q.39:  Five students are participating in a contest.   Three teams are

to be made in such a way that each student has to be a member of

74

Page 75: msd1

one and only one team.  However, team sizes need not be the same.

If the order of the teams or the order of the students within the teams

does not matter, the number of ways in which three teams can be

formed is

 

(A)       35

(B)      7

(C)      20

(D)      25

 

Q.40:  Any complex number x + iy can be put in the form r( cos +

isin) where r is called the modulus and is the argument of the

complex number. The complex number z having the least positive

argument and satisfying  | z – 5i | 3  is

 

(A)       9/5 + i (15/5)

(B)      2/5 + i (3/5)

(C)      12/5 + i (16/5)

(D)      none of the above

 

75

Page 76: msd1

Q. 41: The angle of elevation of a kite from a point 100 meters above

a lake is 30O and the angle of depression of its reflection in the lake is

60O. The height of the kite above the lake is

 

(A)      100 3/3 meters

(B)      200 3 meters

(C)      100 meters

(D)      200 meters

 Section : Data Interpretation

 

 

76

Page 77: msd1

Questions 42 –45 are based on the above graph, which depicts the

performance parameters of LMN Ltd, for ten years. 

 

Q.42: In which of the following years did LMN Ltd. suffer a loss?

 

(A)     1991-92, 1992-93, 1993-94, 1994-95

(B)    1990-91, 1991-92, 1992-93, 1993-94

(C)    1993-94, 1994-95, 1995-96

(D)    1991-92, 1992-93, 1994-95

 

Q.43: The highest percentage of growth in total revenue was recorded

between the financial years

 

(A)     1999-00 and 2000-01

(B)    1997-98 and 1998-99

(C)    1992-93 and 1993-94

(D)    1994-95 and 1995-96

 

77

Page 78: msd1

Q.44: If the percentage growth rate between the financial years 1999-

00 and 2000-01 was maintained, the total revenue for the year 2001-

02 would approximately be

 

(A)     Rs. 192.30 millions.

(B)    Rs. 171.87 millions.

(C)    Rs. 174.74 millions.

(D)    Rs. 164.41 millions.

 

Q.45: In which year was the Net Profit at its highest as a percentage

of total revenue?

 

(A)     1989-90

(B)    2000-01

(C)    1999-00

(D)    1990-91

78

Page 79: msd1

DIRECTIONS

Directions for Problem Solving Questions(Questions 46,47,50,52,53,55)

Solve each problem and indicate the best of the answer choices given.

All numbers used are real numbers.

Directions for Data Sufficiency Questions(Questions 48,49,51,54)

Each Data Sufficiency problem consists of a question and two statements, labeled (1) and (2), in which certain data are given. You have to decide whether the data given in the statements are sufficient for answering the question. Using the data given in the statements plus your knowledge of mathematics and everyday facts, you must indicate whether:

o statement (1) ALONE is sufficient, but statement (2) alone is not sufficient to answer the question asked;

o statement (2) ALONE is sufficient, but statement (1) alone is not sufficient to answer the question asked;

o BOTH statements (1) and (2) TOGETHER are sufficient to answer the question asked; but NEITHER statement ALONE is sufficient;

o EACH statement ALONE is sufficient to answer the question asked;

o statements (1) and (2) TOGETHER are NOT sufficient to answer the question asked, and additional data specific to the problem are needed.

Note: In data sufficiency problems that ask you for the value of a quantity the data given in the statements are sufficient only when it is possible to determine exactly one numerical value for the quantity.

All numbers used are real numbers.

Question 46

If the average cost of producing one widget decreases from $12.50 to $10.75, what is the percent of the decrease?

(A)  10 (B)  12.5

79

Page 80: msd1

(C)  14 (D)  15 (E)  16

Question 47

(A)  1........... (B)  ...... (C) 

(D)  ...... (E)  25

Question 48

Is p + q > pq ?

(1)  p > 0 > q

(2)  |q| = p

(A)  Statement (1) ALONE is sufficient to answer the question, but statement (2) alone is NOT sufficient.

(B)  Statement (2) ALONE is sufficient to answer the question, but statement (1) alone is NOT sufficient.

(C)  BOTH statements (1) and (2) TOGETHER are sufficient to answer the question, but NEITHER statement ALONE is sufficient.

(D)  Each statement ALONE is sufficient to answer the question. (E)  Statements (1) and (2) TOGETHER are NOT sufficient to answer the question.

Question 49

What is the average (arithmetic mean) of x and y? (1) The average (arithmetic mean) of (x + 1) and (y - 1) is equal to the average

(arithmetic mean) of x and y.

(2) The average (arithmetic mean) of x, y, and -1 is 1.

(A)  Statement (1) ALONE is sufficient to answer the question, but statement (2) alone is NOT sufficient.

(B)  Statement (2) ALONE is sufficient to answer the question, but statement (1) alone is NOT sufficient.

80

Page 81: msd1

(C)  BOTH statements (1) and (2) TOGETHER are sufficient to answer the question, but NEITHER statement ALONE is sufficient.

(D)  Each statement ALONE is sufficient to answer the question. (E)  Statements (1) and (2) TOGETHER are NOT sufficient to answer the question.

Question 50

If a building b feet high casts a shadow f feet long, then, at the same time of day, a tree t feet high will cast a shadow how many feet long?

(A)  ...... (B)  ...... (C) 

D)  ..... (E) 

Question 51

What is the value of the integer K? (1) K is a prime number between 40 and 50.

(2) The integer (K + 1) is divisible by exactly two different prime numbers.

(A)  Statement (1) ALONE is sufficient to answer the question, but statement (2) alone is NOT sufficient.

(B)  Statement (2) ALONE is sufficient to answer the question, but statement (1) alone is NOT sufficient.

(C)  BOTH statements (1) and (2) TOGETHER are sufficient to answer the question, but NEITHER statement ALONE is sufficient.

(D)  Each statement ALONE is sufficient to answer the question. (E)  Statements (1) and (2) TOGETHER are NOT sufficient to answer the question.

Question 52

If x, y, and z are consecutive non-zero integers, and if x < y < z, which of the following must be a positive odd integer?

(A)  x - y - z (B)  xy + z (C)  x + y - z (D)  xz - y (E)  x + y + z

81

Page 82: msd1

Question 53

Two canoe riders must be selected from each of two groups of campers. One group consists of three men and one woman, and the other group consists of two women and one man. What is the probability that two men and two women will be selected?

(A)  ...... (B)  ...... (C) 

(D)  ....... (E) 

Question 54

At a garage sale, Jeff sold 80% of his books, which include both hardbacks and paperbacks. He sold an equal number of each of the two types of books, selling all paperbacks for $1 each and all hardbacks for $3 each. How many books did Jeff own before the sale? (1) Jeff's total revenue from the sale of his books was $32.

(2) Exactly two hardbacks and two paperbacks remained unsold after the sale.

(A)  Statement (1) ALONE is sufficient to answer the question, but statement (2) alone is NOT sufficient.

(B)  Statement (2) ALONE is sufficient to answer the question, but statement (1) alone is NOT sufficient.

C)  BOTH statements (1) and (2) TOGETHER are sufficient to answer the question, but NEITHER statement ALONE is sufficient.

D)  Each statement ALONE is sufficient to answer the question. (E)  Statements (1) and (2) TOGETHER are NOT sufficient to answer the question.

Question 55

What is the greatest value of a positive integer n such that 3n is a factor of 1815 ?

(A)  15 (B)  18 (C)  30 (D)  33 (E)  45

82

Page 83: msd1

Answer Key

To return to a question, click on the answer.

46. (C)

47. (B)

48. (C) 49. (B) 50. (A) 51. (E) 52. (D) 53. (E) 54. (D) 55. (C)

LECTURE 10-15

DIRECTIONS

Directions for Problem Solving Questions(Questions 1, 2, 5, 7, 8, and 10)

Solve each problem and indicate the best of the answer choices

83

Page 84: msd1

given.

All numbers used are real numbers.

Directions for Data Sufficiency Questions(Questions 3, 4, 6, and 9)

Each Data Sufficiency problem consists of a question and two statements, labeled (1) and (2), in which certain data are given. You have to decide whether the data given in the statements are sufficient for answering the question. Using the data given in the statements plus your knowledge of mathematics and everyday facts, you must indicate whether:

o statement (1) ALONE is sufficient, but statement (2) alone is not sufficient to answer the question asked;

o statement (2) ALONE is sufficient, but statement (1) alone is not sufficient to answer the question asked;

o BOTH statements (1) and (2) TOGETHER are sufficient to answer the question asked; but NEITHER statement ALONE is sufficient;

o EACH statement ALONE is sufficient to answer the question asked;

o statements (1) and (2) TOGETHER are NOT sufficient to answer the question asked, and additional data specific to the problem are needed.

Note: In data sufficiency problems that ask you for the value of a quantity the data given in the statements are sufficient only when it is possible to determine exactly one numerical value for the quantity.

All numbers used are real numbers.

 Question 1

If a + b - c = d, and if a - b + c = e, then a =

(A)  (d + e) (B)  d - e (C)  2d + e (D)  d + e (E)  2(d + e)

84

Page 85: msd1

Question 2

Five years ago, Beth's age was three times that of Amy. Ten years ago, Beth's age was one half that of Chelsea. If C represents Chelsea's current age, which of the following represents Amy's current age?

(A)  ...... (B)  2C ...... (C) 

(D)  3C – 5 ...... (E) 

Question 3

If the city of Franklin lies along a certain route between the cities of Maple Valley and Viewcrest, what is the distance along the route from Franklin to Maple Valley? (1) Traveling from Viewcrest to Maple Valley at an average rate of 45 m.p.h.

takes 20 minutes longer than at an average rate of 50 m.p.h.

(2) Traveling along the route at a constant 25 miles per gallon of gas, a car will use twice as much gas between Maple Valley and Franklin as it will use between Franklin and Viewcrest.

(A)  Statement (1) ALONE is sufficient to answer the question, but statement (2) alone is NOT sufficient.

(B)  Statement (2) ALONE is sufficient to answer the question, but statement (1) alone is NOT sufficient.

(C)  BOTH statements (1) and (2) TOGETHER are sufficient to answer the question, but NEITHER statement ALONE is sufficient.

(D)  Each statement ALONE is sufficient to answer the question.E)  Statements (1) and (2) TOGETHER are NOT sufficient to answer the question.

Question 4

If A, B, C, and D are all positive numbers, is the value of A – B greater than the value of C – D? (1) A + D = B + C

(2) A and B are each greater in value than either C or D.

(A)  Statement (1) ALONE is sufficient to answer the question, but statement (2)

85

Page 86: msd1

alone is NOT sufficient. (B)  Statement (2) ALONE is sufficient to answer the question, but statement (1)

alone is NOT sufficient. (C)  BOTH statements (1) and (2) TOGETHER are sufficient to answer the

question, but NEITHER statement ALONE is sufficient. (D)  Each statement ALONE is sufficient to answer the question. (E)  Statements (1) and (2) TOGETHER are NOT sufficient to answer the question.

Question 5

A portion of $6600 is invested at a 5% annual return, while the remainder is invested at a 3% annual return. If the annual income from the portion earning a 5% return is twice that of the other portion, what is the total income from the two investments after one year?

(A) $180 (B)  $270 (C)  $300 (D)  $320 (E)  $360

(B)

Question 6

Of all sophomore students attending a certain college, 20% are enrolled in an English course while 10% are enrolled in a psychology course. How many sophomore students attend the college? (1) 5% of the sophomore students at the college are enrolled in both an English

course and a psychology course.

(2) A total of 210 sophomore students at the college are enrolled in neither an English course nor a psychology course.

(A)  Statement (1) ALONE is sufficient to answer the question, but statement (2) alone is NOT sufficient.

(B)  Statement (2) ALONE is sufficient to answer the question, but statement (1) alone is NOT sufficient.

(C)  BOTH statements (1) and (2) TOGETHER are sufficient to answer the question, but NEITHER statement ALONE is sufficient.

(D)  Each statement ALONE is sufficient to answer the question. (E)  Statements (1) and (2) TOGETHER are NOT sufficient to answer the question.

86

Page 87: msd1

Question 7

If one typist can type a certain manuscript in 8 minutes, while another typist can type the same manuscript in 12 minutes, how long would it take both typists, working simultaneously, to type the manuscript?

(A)  3 minutes, 36 seconds (B)  4 minutes, 48 seconds (C)  5 minutes, 15 seconds (D)  6 minutes (E)  10 minutes

Question 8

If r = and s = p – q, for which of the following values of p would r2 = s2 ?

(A)  ...... (B)  10 – ...... (C)  q – 1

(D)  3q ........ (E)  – 9

Question 9

What is the value of x?

(1)x + y = 3

(2)y + x = 2

(A)  Statement (1) ALONE is sufficient to answer the question, but statement (2) alone is NOT sufficient.

(B)  Statement (2) ALONE is sufficient to answer the question, but statement (1) alone is NOT sufficient.

(C)  BOTH statements (1) and (2) TOGETHER are sufficient to answer the question, but NEITHER statement ALONE is sufficient.

(D)  Each statement ALONE is sufficient to answer the question. (E)  Statements (1) and (2) TOGETHER are NOT sufficient to answer the question.

87

Page 88: msd1

Question 10

If eight pounds of macadamia nuts, priced at $6.00 per pound, are combined with twelve pounds of brazil nuts, priced at $5.00 per pound, what is the per-pound price of the resulting mixture?

(A)  $5.25 (B)  $5.40 (C)  $5.50 (D)  $5.75 (E)  $5.80

Answer Key

To return to a question, click on the answer. 1. (A) 2. (A) 3. (C) 4. (A) 5. (B) 6. (C) 7. (B) 8. (A) 9. (E) 10. (B)

OFFICIAL TEST DIRECTIONS

Directions for Problem Solving Questions(Questions 11,12,15,17,18, and 20)

Solve each problem and indicate the best of the answer choices given.

All numbers used are real numbers.

Directions for Data Sufficiency Questions(Questions 13, 14, 16, and1 9)

Each Data Sufficiency problem consists of a question and two statements, labeled (1) and (2), in which certain data are given.

88

Page 89: msd1

You have to decide whether the data given in the statements are sufficient for answering the question. Using the data given in the statements plus your knowledge of mathematics and everyday facts, you must indicate whether:

o statement (1) ALONE is sufficient, but statement (2) alone is not sufficient to answer the question asked;

o statement (2) ALONE is sufficient, but statement (1) alone is not sufficient to answer the question asked;

o BOTH statements (1) and (2) TOGETHER are sufficient to answer the question asked; but NEITHER statement ALONE is sufficient;

o EACH statement ALONE is sufficient to answer the question asked;

o statements (1) and (2) TOGETHER are NOT sufficient to answer the question asked, and additional data specific to the problem are needed.

Note: In data sufficiency problems that ask you for the value of a quantity the data given in the statements are sufficient only when it is possible to determine exactly one numerical value for the quantity.

All numbers used are real numbers.

 Question 11

....

If AB and AC, inscribed in a square as shown above, are each 10 centimeters in length, what is the area of the square (in centimeters)?

(A)  (B)  74 (C)  80

(D)  (E)  100

89

Page 90: msd1

Question 12

If a rectangle's length and width are both doubled, by what percent is the rectangle's area increased?

(A)  50 (B)  100 (C)  200 (D)  300 (E)  400

Question 13

....

In the figure above, is ?

(1) x + y = 180

(2) w = z

(A)  Statement (1) ALONE is sufficient to answer the question, but statement (2) alone is NOT sufficient.

(B)  Statement (2) ALONE is sufficient to answer the question, but statement (1) alone is NOT sufficient.

(C)  BOTH statements (1) and (2) TOGETHER are sufficient to answer the question, but NEITHER statement ALONE is sufficient.

(D)  Each statement ALONE is sufficient to answer the question. (E)  Statements (1) and (2) TOGETHER are NOT sufficient to answer the question.

Question 14

90

Page 91: msd1

....

What is the value of x in the figure above?

(1) y + z = 140

(2) w + x = 80

(A)  Statement (1) ALONE is sufficient to answer the question, but statement (2) alone is NOT sufficient.

(B)  Statement (2) ALONE is sufficient to answer the question, but statement (1) alone is NOT sufficient.

(C)  BOTH statements (1) and (2) TOGETHER are sufficient to answer the question, but NEITHER statement ALONE is sufficient.

(D)  Each statement ALONE is sufficient to answer the question. (E)  Statements (1) and (2) TOGETHER are NOT sufficient to answer the question.

Question 15

In a certain parallelogram, the degree measure of one angle exceeds that of another angle by 30. What is the degree measure of the smaller angle?

(A)  30 (B)  55 (C)  60 (D)  70 (E)  75

Question 16

80 linear feet of fencing material is used to construct a rectangular enclosure. What is the area of the enclosure? (1) The length of one particular side of the enclosure is 10 feet.

(2) It is possible to divide the enclosure into three square enclosures of equal size.

(A)  Statement (1) ALONE is sufficient to answer the question, but statement (2) alone is NOT sufficient.

91

Page 92: msd1

(B)  Statement (2) ALONE is sufficient to answer the question, but statement (1) alone is NOT sufficient.

(C)  BOTH statements (1) and (2) TOGETHER are sufficient to answer the question, but NEITHER statement ALONE is sufficient.

(D)  Each statement ALONE is sufficient to answer the question. (E)  Statements (1) and (2) TOGETHER are NOT sufficient to answer the question.

Question 17

On the xy-coordinate plane, all of the following xy-coordinate points lie on the circumference of a circle whose radius is 10 and whose center lies at the (x,y) point (-1,0) EXCEPT:

(A)  (–6, –7)

(B)  (–2, ) (C)  (–1, –10) (D)  (7, 6)

(E)  (1, – )

Question 18

....

Note: Figure not drawn to scale...

Point Q lies at the center of the square base (ABCD) of the pyramid pictured above. The pyramid's height (PQ) measures exactly one half the length of each edge of its base, and point E lies exactly halfway between C and D along one edge of the base. What is the ratio of the surface area of any of the pyramid's four triangular faces to the surface area of the shaded triangle?

(A)  3 : ......... (B)  :1....... (C)  :3

92

Page 93: msd1

(D)  2 :1........... (E)  8 :

Question 19

....

In the figure above, does x = 90?

(1) The length of AC is less than the length of BC.

(2) The length of AB is one-fourth the circumference of the circle.

(A)  Statement (1) ALONE is sufficient to answer the question, but statement (2) alone is NOT sufficient.

(B)  Statement (2) ALONE is sufficient to answer the question, but statement (1) alone is NOT sufficient.

(C)  BOTH statements (1) and (2) TOGETHER are sufficient to answer the question, but NEITHER statement ALONE is sufficient.

(D)  Each statement ALONE is sufficient to answer the question. (E)  Statements (1) and (2) TOGETHER are NOT sufficient to answer the question.

Question 20

How many complete tanks of water, each with a capacity of 3 cubic meters, are needed to fill an empty cylindrical tank whose height is 3 meters and whose base has a radius of 2 meters?

(A)  12 (B)  13 (C)  14 (D)  15 (E)  16

Answer Key

To return to a question, click on the answer. 1. (C) 2. (D)

93

Page 94: msd1

3. (C) 4. (D) 5. (E) 6. (D) 7. (A) 8. (D) 9. (B) 10. (B)

DIRECTIONS

Directions for Problem Solving Questions

Solve each problem and indicate the best of the answer choices given.

All numbers used are real numbers.

 Questions21 and 22 refer to the following table:

Question 21

How much more does it cost to send a parcel weighing 16.5 pounds by express delivery than to send the same parcel by ground delivery?

(A)  $3.45 (B)  $9.00 (C)  $11.55 (D)  $15.00 (E)  $16.60 

94

Page 95: msd1

Question 22

By approximately what percent does the average air delivery cost per pound for a 21-pound parcel exceed the average ground delivery cost per pound for a 28-pound parcel?

(A)  33 (B)  46 (C)  67 (D)  80 (E)  130

Questions 23 and 24 refer to the following graph:

Question 23

Marketing expenses trailed production expenses by about the same percentage during which of the following two years?

(A)  1990 and 1992 (B)  1991 and 1992 (C)  1991 and 1993 (D)  1992 and 1993 (E)  1990 and 1994 

Question 24

Consider aggregate legal and production expenses for each year shown. With respect to the two years during which this aggregate amount was most nearly the same, average annual marketing expenses totaled approximately

95

Page 96: msd1

(A)  $5.5 million (B)  $6.1 million (C)  $6.5 million (D)  $7.2 million (E)  $8.0 million

Answer Key

To return to a question, click on the answer. 21. (D)22. (C) 23. (A) 24. (B)

96